Difference between revisions of "1983 IMO Problems/Problem 2"
(Created page with "Let <math>A</math> be one of the two distinct points of intersection of two unequal coplanar circles <math>C1</math> and <math>C2</math> with centers <math>O1</math> and <math...") |
|||
(2 intermediate revisions by one other user not shown) | |||
Line 1: | Line 1: | ||
− | Let <math>A</math> be one of the two distinct points of intersection of two unequal coplanar circles <math> | + | ==Problem== |
+ | Let <math>A</math> be one of the two distinct points of intersection of two unequal coplanar circles <math>C_1</math> and <math>C_2</math> with centers <math>O_1</math> and <math>O_2</math> respectively. One of the common tangents to the circles touches <math>C_1</math> at <math>P_1</math> and <math>C_2</math> at <math>P_2</math>, while the other touches <math>C_1</math> at <math>Q_1</math> and <math>C_2</math> at <math>Q_2</math>. Let <math>M_1</math> be the midpoint of <math>P_1Q_1</math> and <math>M_2</math> the midpoint of <math>P_2Q_2</math>. Prove that <math>\angle O_1AO_2=\angle M_1AM_2</math>. | ||
− | {{IMO box|year=1983| | + | ==Solution 1== |
+ | Let <math>A</math> be one of the two distinct points of intersection of two unequal coplanar circles <math>C_1</math> and <math>C_2</math> with centers <math>O_1</math> and <math>O_2</math> respectively. Let <math>S</math> be such point on line <math>O_1O_2</math> so that tangents on <math>C_1</math> touches it at <math>P_1</math> and <math>Q_1</math> and tangents on <math>C_2</math> touches it at <math>P_2</math> and <math>Q_2</math>. Let <math>M_1</math> be the midpoint of <math>P_1Q_1</math> and <math>M_2</math> the midpoint of <math>P_2Q_2</math>. Prove that <math>\angle O_1AO_2 = \angle M_1AM_2</math>. | ||
+ | |||
+ | Proof: Since <math>S</math> is image of <math>M_1</math> under inversion wrt circle <math>C_1</math> we have:<cmath> \angle O_1AM_1 = \angle O_1M_1'A'= \angle O_1SA </cmath>Since <math>S</math> is image of <math>M_2</math> under inversion wrt circle <math>C_2</math> we have:<cmath> \angle O_2SA= \angle O_2A'S'= \angle O_2AM_2 </cmath>Image of <math>A</math> is in both cases <math>A</math> itself, since it lies on both circles. | ||
+ | Since <math>\angle O_1SA = \angle O_2SA</math> we have:<cmath> \angle M_1AO_1=\angle M_2AO_2 </cmath>Now:<cmath> \angle O_1AO_2 = \angle M_1AM_2-\angle M_1AO_1+\angle M_2AO_2 = \angle M_1AM_2 </cmath> | ||
+ | |||
+ | This solution was posted and copyrighted by Number1. The original thread for this problem can be found here: [https://aops.com/community/p444724] | ||
+ | |||
+ | ==Solution 2== | ||
+ | Let <math> P_1P_2</math> and <math> Q_1Q_2</math> meet at <math> R</math>. Let <math> RA</math> meet <math> C_2</math> at <math> B</math>. Now, it is well-known that <math> O_1O_2</math>, <math> P_1P_2</math>, and <math> Q_1Q_2</math> are concurrent at <math> R</math>, the center of homothety between <math> C_1</math> and <math> C_2</math>. Now, it is well-known that <math> O_1O_2</math> bisects <math> \angle P_1RQ_1</math>. Since <math> P_1R = Q_1R</math>, we have that <math> O_1O_2R</math> meets <math> P_1Q_1</math> at its midpoint, <math> M_1</math>, and <math> P_1Q_1</math> is perpendicular to <math> O_1O_2R</math>. Similarly, <math> O_1O_2R</math> passes through <math> M_2</math> and is perpendicular to <math> P_2Q_2</math>. Since <math> O_2P_2\perp P_2R</math>, we have that <math> RA\cdot RB = RP_2^2 = RM_2\cdot RO_2</math>, which implies that <math> ABM_2O_2</math> is cyclic. Yet, since <math> A</math> and <math> B</math> lie on <math> C_1</math> and <math> C_2</math> respectively and are collinear with <math> R</math>, we see that the homothety that maps <math> C_1</math> to <math> C_2</math> about <math> R</math> maps <math> A</math> to <math> B</math>. Also, <math> O_1</math> is mapped to <math> O_2</math> by this homothety, and since <math> M_1</math> and <math> M_2</math> are corresponding parts in these circles, <math> M_1</math> is mapped to <math> M_2</math> by this homothety, so <math> \angle O_1AM_1 = \angle O_2BM_2 = \angle O_2AM_2</math>, from which we conclude that <math> \angle O_2AO_1 = \angle M_2AOM_1</math>. | ||
+ | |||
+ | This solution was posted and copyrighted by The QuattoMaster 6000. The original thread for this problem can be found here: [https://aops.com/community/p1247276] | ||
+ | |||
+ | ==Solution 3== | ||
+ | Let <math>B</math> be the other intersection point of these two circles. Let <math>P_1P_2</math>, <math>Q_1Q_2</math>, <math>O_1O_2</math> meet at <math>Q</math>. Let <math>AB</math> meet <math>P_1P_2</math> at <math>P</math>. | ||
+ | Clearly, <math>P_1Q_1</math> and <math>O_1O_2</math> are perpendicular at <math>M_1</math>; <math>P_2Q_2</math> and <math>O_1O_2</math> are perpendicular at <math>M_2</math>. | ||
+ | Since <math>\triangle O_1P_1M_1 \sim \triangle O_2P_2M_2</math>,<cmath>\dfrac{O_1P_1}{O_1M_1} = \dfrac{O_2P_2}{O_2M_2} \Rightarrow \dfrac{O_1A}{O_1M_1} = \dfrac{O_2A}{O_2M_2} \qquad{(*)}</cmath>Since <math>P</math> is on the radical axis, <math>PP_1 = PP_2</math>, so in the right trapezoid <math>P_1P_2M_2M_1</math>, <math>AB</math> is the midsegment. So we have <math>M_1A = AM_2</math> and <math>\angle AM_1O_1 = \angle AM_2O_1</math>. | ||
+ | Let <math>M</math> be a point on <math>O_1O_2</math> such that <math>\triangle AM_1O_1 \cong \triangle AM_2M</math> (which means <math>AM=AO_1</math> ve <math>MM_2 = M_1O_1</math>). So, from <math>(*)</math>, we get<cmath>\dfrac{AM}{MM_2} = \dfrac{O_2A}{O_2M_2}</cmath>This means, <math>AM_2</math> is the angle bisector of <math>\angle MAO_2</math>. So,<cmath>\angle O_2AM_2 = \angle M_2AM= \angle M_1AO_1 \Longrightarrow \angle M_1AM_2 = \angle O_1AO_2.</cmath> | ||
+ | |||
+ | This solution was posted and copyrighted by matematikolimpiyati. The original thread for this problem can be found here: [https://aops.com/community/p3260115] | ||
+ | |||
+ | ==Solution 4== | ||
+ | Let, <math>P_1P_2,Q_1Q_2,O_1O_2</math> concur at <math>Q</math>.Then a homothety with centre <math>Q</math> that sends <math>C_1</math> to <math>C_2</math>.Let <math>QA \cap C_1 =B</math>.Under the homothety <math>A</math> is the image of <math>B</math>.So, <math>\angle M_1BO_1 =\angle M_2AO_2</math> and <math>\triangle QP_1O_1 \sim \triangle QM_1P_1 \implies \frac{QO_{1}}{QP_{1}} = \frac{QP_{1}}{QM_{1}}</math>.so <math>QO_1.QM_1 = Q.P^2_1 = QA .QB</math> Hence <math>A,M_1,B,O_1</math> are concyclic.so <math>\angle O_1BM_1=\angle O_1AM_1 \implies \angle O_1AM_1= \angle O_2AM_2 \implies \angle O_1AO_2=\angle M_1AM_2 </math> | ||
+ | |||
+ | This solution was posted and copyrighted by spikerboy. The original thread for this problem can be found here: [https://aops.com/community/p3478185] | ||
+ | |||
+ | ==Solution 5== | ||
+ | It can be easily solved with this lemma: | ||
+ | Let <math>A</math>-symmedian of <math>\triangle ABC</math> intersect its circumscribed circle <math>u</math> at <math>D, P</math> on <math>AD</math> satisfying <math>AP = PD</math>. Let us define center of <math>u</math> as <math>O</math>. Then <math>(B, C, P, O)</math> are concyclic. | ||
+ | Now let <math>S</math> be intersection of tangents to circles from the problem, <math>AS</math> intersects <math> C_1</math> at <math>B</math>. One can prove that <math>\square AP_1BQ_1</math> is harmonic qudrilateral, so <math>P_1Q_1</math> is symmedian of <math>\triangle AP_1B</math>. By lemma we get that <math>(B, A, O_1, M_1)</math> are concyclic, thus <math>\angle O_1AM_1</math> is equivalent to <math>\angle O_1BM_1</math>. By homothety we get <math>\angle O_1BM_1 = \angle O_2AM_2</math>. | ||
+ | <math>Q. E. D.</math> | ||
+ | |||
+ | This solution was posted and copyrighted by LeoLeon. The original thread for this problem can be found here: [https://aops.com/community/p18853122] | ||
+ | |||
+ | |||
+ | |||
+ | {{IMO box|year=1983|num-b=1|num-a=3}} |
Latest revision as of 22:35, 29 January 2021
Problem
Let be one of the two distinct points of intersection of two unequal coplanar circles and with centers and respectively. One of the common tangents to the circles touches at and at , while the other touches at and at . Let be the midpoint of and the midpoint of . Prove that .
Solution 1
Let be one of the two distinct points of intersection of two unequal coplanar circles and with centers and respectively. Let be such point on line so that tangents on touches it at and and tangents on touches it at and . Let be the midpoint of and the midpoint of . Prove that .
Proof: Since is image of under inversion wrt circle we have:Since is image of under inversion wrt circle we have:Image of is in both cases itself, since it lies on both circles. Since we have:Now:
This solution was posted and copyrighted by Number1. The original thread for this problem can be found here: [1]
Solution 2
Let and meet at . Let meet at . Now, it is well-known that , , and are concurrent at , the center of homothety between and . Now, it is well-known that bisects . Since , we have that meets at its midpoint, , and is perpendicular to . Similarly, passes through and is perpendicular to . Since , we have that , which implies that is cyclic. Yet, since and lie on and respectively and are collinear with , we see that the homothety that maps to about maps to . Also, is mapped to by this homothety, and since and are corresponding parts in these circles, is mapped to by this homothety, so , from which we conclude that .
This solution was posted and copyrighted by The QuattoMaster 6000. The original thread for this problem can be found here: [2]
Solution 3
Let be the other intersection point of these two circles. Let , , meet at . Let meet at . Clearly, and are perpendicular at ; and are perpendicular at . Since ,Since is on the radical axis, , so in the right trapezoid , is the midsegment. So we have and . Let be a point on such that (which means ve ). So, from , we getThis means, is the angle bisector of . So,
This solution was posted and copyrighted by matematikolimpiyati. The original thread for this problem can be found here: [3]
Solution 4
Let, concur at .Then a homothety with centre that sends to .Let .Under the homothety is the image of .So, and .so Hence are concyclic.so
This solution was posted and copyrighted by spikerboy. The original thread for this problem can be found here: [4]
Solution 5
It can be easily solved with this lemma: Let -symmedian of intersect its circumscribed circle at on satisfying . Let us define center of as . Then are concyclic. Now let be intersection of tangents to circles from the problem, intersects at . One can prove that is harmonic qudrilateral, so is symmedian of . By lemma we get that are concyclic, thus is equivalent to . By homothety we get .
This solution was posted and copyrighted by LeoLeon. The original thread for this problem can be found here: [5]
1983 IMO (Problems) • Resources | ||
Preceded by Problem 1 |
1 • 2 • 3 • 4 • 5 • 6 | Followed by Problem 3 |
All IMO Problems and Solutions |